If x,y,z are each greater than 1, show that
x4(y−1)2+y4(z−1)2+z4(x−1)2≥48\frac { { x }^{ 4 } }{ { (y-1) }^{ 2 } } +\frac { { y }^{ 4 } }{ { (z-1 })^{ 2 } } +\frac { { z }^{ 4 } }{ { (x-1) }^{ 2 } } \ge 48(y−1)2x4+(z−1)2y4+(x−1)2z4≥48
This a part of my set NMTC 2nd Level (Junior) held in 2014.
Note by Siddharth G 6 years, 7 months ago
Easy Math Editor
This discussion board is a place to discuss our Daily Challenges and the math and science related to those challenges. Explanations are more than just a solution — they should explain the steps and thinking strategies that you used to obtain the solution. Comments should further the discussion of math and science.
When posting on Brilliant:
*italics*
_italics_
**bold**
__bold__
- bulleted- list
1. numbered2. list
paragraph 1paragraph 2
paragraph 1
paragraph 2
[example link](https://brilliant.org)
> This is a quote
This is a quote
# I indented these lines # 4 spaces, and now they show # up as a code block. print "hello world"
\(
\)
\[
\]
2 \times 3
2^{34}
a_{i-1}
\frac{2}{3}
\sqrt{2}
\sum_{i=1}^3
\sin \theta
\boxed{123}
Firstly, we have, (a−2)2≥0 ⟹ a2−4a+4≥0 ⟹ a2≥4(a−1) ⟹ a2a−1≥4 ⟹ a4(a−1)2≥16 (a-2)^2 \geq 0 \implies a^2 -4a + 4 \geq 0 \implies a^2 \geq 4(a-1) \implies \frac{a^2}{a-1} \geq 4 \implies \frac{a^4}{(a-1)^2} \geq 16 (a−2)2≥0⟹a2−4a+4≥0⟹a2≥4(a−1)⟹a−1a2≥4⟹(a−1)2a4≥16
Now, By AM-GM,
x4(y−1)2+y4(z−1)2+z4(x−1)2≥3x4(y−1)2∗y4(z−1)2∗z4(x−1)23=3x4(x−1)2∗y4y−1)2∗z4(z−1)23≥316∗16∗163=3∗16=48. \frac{x^4}{(y-1)^2} + \frac{y^4}{(z-1)^2} + \frac{z^4}{(x-1)^2} \geq 3\sqrt[3]{\frac{x^4}{(y-1)^2}* \frac{y^4}{(z-1)^2}* \frac{z^4}{(x-1)^2} } = 3\sqrt[3]{\frac{x^4}{(x-1)^2}* \frac{y^4}{y-1)^2}* \frac{z^4}{(z-1)^2} } \geq 3\sqrt[3]{16*16*16} = 3 * 16 = 48. (y−1)2x4+(z−1)2y4+(x−1)2z4≥33(y−1)2x4∗(z−1)2y4∗(x−1)2z4=33(x−1)2x4∗y−1)2y4∗(z−1)2z4≥3316∗16∗16=3∗16=48.
Log in to reply
Amazing answer! Thank you!
Is the AM-GM step necessary? You could just say x4(x−1)4≥16\frac{x^4}{(x-1)^4} \geq 16(x−1)4x4≥16 and so on for y and z and add the 3 inequalities together right?
@Josh Banister – But in the question, the denominator and the numerator are of different variables, which makes the AM-GM necessary to bring the denominator and the numerator with the same variables together. Thus x4(x−1)2≥16\frac { { x }^{ 4 } }{ { (x-1) }^{ 2 } } \ge 16(x−1)2x4≥16 can only be used after the AM_GM step.
@Siddharth G – You're right. Just skipped over that for some reason ^o^
Problem Loading...
Note Loading...
Set Loading...
Easy Math Editor
This discussion board is a place to discuss our Daily Challenges and the math and science related to those challenges. Explanations are more than just a solution — they should explain the steps and thinking strategies that you used to obtain the solution. Comments should further the discussion of math and science.
When posting on Brilliant:
*italics*
or_italics_
**bold**
or__bold__
paragraph 1
paragraph 2
[example link](https://brilliant.org)
> This is a quote
\(
...\)
or\[
...\]
to ensure proper formatting.2 \times 3
2^{34}
a_{i-1}
\frac{2}{3}
\sqrt{2}
\sum_{i=1}^3
\sin \theta
\boxed{123}
Comments
Firstly, we have, (a−2)2≥0⟹a2−4a+4≥0⟹a2≥4(a−1)⟹a−1a2≥4⟹(a−1)2a4≥16
Now, By AM-GM,
(y−1)2x4+(z−1)2y4+(x−1)2z4≥33(y−1)2x4∗(z−1)2y4∗(x−1)2z4=33(x−1)2x4∗y−1)2y4∗(z−1)2z4≥3316∗16∗16=3∗16=48.
Log in to reply
Amazing answer! Thank you!
Log in to reply
Is the AM-GM step necessary? You could just say (x−1)4x4≥16 and so on for y and z and add the 3 inequalities together right?
Log in to reply
(x−1)2x4≥16 can only be used after the AM_GM step.
But in the question, the denominator and the numerator are of different variables, which makes the AM-GM necessary to bring the denominator and the numerator with the same variables together. ThusLog in to reply